Finding Convergence Radius & Interval: Solving a Complex Homework Problem

Click For Summary
The discussion centers on finding the radius and interval of convergence for the series ∑((-1)^n (x+3)^(3n)/(2n ln n)). The Ratio Test was applied, leading to a limit of -1/2, which caused confusion regarding the negative radius of convergence. Participants highlighted the importance of maintaining absolute values and correctly applying limits, specifically noting the missing factor of 2 in the denominator. Clarifications on handling negative values helped resolve previous issues with convergence. The conversation emphasizes the need for careful application of mathematical principles in series convergence problems.
Grunting7
Messages
2
Reaction score
0

Homework Statement



n=3 ∑ ((-1)n (x+3)3n)/(2nlnn)

Find radius of convergence, interval of convergence, values for x which series is: absolutely convergent, conditionally converge or divergence.

Homework Equations

The Attempt at a Solution


I applied the Ratio Test and got

|(x+3)3| lim n--> ∞ (-1(lnn))/(ln(n+1))

Then I used l'Hospitals to get the limit and got -1/2.
So then it's -1/2 * |(x+3)3| = L. Then do the radius and interval stuff.

The problem is that it's -1/2 and the radius can't be negative. I've had one or two similar problems where I keep getting a negative radius. Not sure what I am missing.
 
Physics news on Phys.org
The absolute value of -1 is 1. When you're pulling out the (x+3)^3, you have to keep the absolute value on the lnn/ln(n+1) or pull out the -1 with the (x+3)^3. Also I am pretty sure you're missing (1/2) somewhere in your limit.
 
  • Like
Likes Grunting7
Wow, did not even know that. Solves the negative radius I was having with the other problems.
Thanks alot!

Yea, I forgot the 2 in the denominator of the limit.
 
Question: A clock's minute hand has length 4 and its hour hand has length 3. What is the distance between the tips at the moment when it is increasing most rapidly?(Putnam Exam Question) Answer: Making assumption that both the hands moves at constant angular velocities, the answer is ## \sqrt{7} .## But don't you think this assumption is somewhat doubtful and wrong?

Similar threads

  • · Replies 2 ·
Replies
2
Views
2K
  • · Replies 4 ·
Replies
4
Views
1K
  • · Replies 5 ·
Replies
5
Views
2K
  • · Replies 24 ·
Replies
24
Views
3K
  • · Replies 2 ·
Replies
2
Views
2K
Replies
2
Views
1K
  • · Replies 7 ·
Replies
7
Views
2K
  • · Replies 5 ·
Replies
5
Views
2K
  • · Replies 2 ·
Replies
2
Views
1K
  • · Replies 11 ·
Replies
11
Views
3K